Answer:
67.9m2
Step-by-step explanation:
HELP!!
Which Platonic solid has four faces that are equilateral triangles?
A. Hexahedron
B. Dodecahedron
C. Tetrahedron
D. Icosahedron
Answer:
C. Tetrahedron
Step-by-step explanation:
In figure
The Platonic solid that has four faces that are equilateral triangles is the Tetrahedron. The correct option is C.
What is Tetrahedron?A Tetrahedron is a three-dimensional shape made up of four equilateral triangles. It is one of the five Platonic solids, which are regular, convex polyhedra with identical vertices and faces.
Each vertex in a Tetrahedron is connected to the other three vertices by an edge, resulting in a pyramid-like structure.
The Tetrahedron is a unique shape with numerous applications in mathematics, engineering, and chemistry.
The Tetrahedron is a Platonic solid with four equilateral triangle faces. It has four faces, four vertices, and six edges in total. It is the most basic of the Platonic solids and the only one with no parallel faces.
Thus, the correct option is C.
For more details regarding Tetrahedron, visit:
https://brainly.com/question/11946461
#SPJ7
Decimal points are sometimes used in hotel room numbers. If you were assigned to room 6.10, where you expect to find it?
Answer:
the 10th room on the 6th floor
Step-by-step explanation:
I would guess that the 6 would be the floor number since is a whole number than the 10 would be the room on said floor because it after the 6.
If you were assigned to room 6.10, you would expect to find it in 6th floor and room number 10.
What is decimal?Decimals are numbers that have two components, a whole number component and a fractional component, which are separated by a decimal point.
Given:
Decimal points are sometimes used in hotel room numbers.
Any decimal number has whole part and fractional part.
Then whole part = number of floor.
Fractional part = number of rooms.
Then if number is 6.10,
then 6th floor and room number 10.
Therefore, 6th floor and room number 10.
To learn more about the decimal;
brainly.com/question/109147
#SPJ2
the yearbook club is handing out t-shirts to its members there are five blue Seven Green 9 red and 4 yellow t-shirts in all. if Jacob is handed a t-shirt what is the probability that the color is red
a. 9/25
b. 9/16
c. 3/5
d. 16/25
Answer:
a. 9/25
Step-by-step explanation:
A probability is the number of desired outcomes divided by the number of total outcomes.
In this question:
There are 5 + 7 + 9 + 4 = 25 shirts, 9 of which are red.
So 9/25 probability of being handed a red t-shirt, which means that the correct answer is given by option a.
Find the surface area of the number cube below.
Answer:
Surface area = 24 in.²
Step-by-step explanation:
Surface area of the cube = 6a²
Where,
a = length of the each side of the cube = 2 in.
Plug in the value into the formula
Surface area of the cube = 6(2²)
= 6 × 4
Surface area = 24 in.²
Tome los datos de la figura y escriba los valores de my b
Answer:
m=1 y b=-2
Step-by-step explanation:
Ecuación de la recta
La recta puede escribirse de varias formas. Una de ellas es la llamada punto-pendiente, donde construimos la ecuación conociendo estas dos variables.
La ecuación punto-pendiente se expresa de la siguiente forma:
y - yo = m( x - xo )
Donde m es la pendiente y (xo,yo) es el punto por donde la recta pasa.
Para calcular la pendiente usamos la fórmula siguiente:
\(\displaystyle m=\frac{y_2-y_1}{x_2-x_1}\)
Donde (x1,y1) y (x2,y2) son dos puntos de la recta.
Del gráfico extraemos los puntos (0,-2) y (4,2), entonces:
\(\displaystyle m=\frac{2+2}{4-0}=\frac{4}{4}=1\)
La pendiente m es igual a 1.
El parámetro b se conoce como la ordenada en el origen, y es el lugar donde la recta cruza el eje y.
Podemos ver claramente en el gráfico que eso ocurre cuando y=-2.
En resumen, m=1 y b=-2
825 use each digit once. make the smallest 3digit number
Step-by-step explanation:
Given: To make smallest 3-digit number of 825.
To find: The smallest 3-digit number of 825.
Solution: We can make the smallest 3-digit number of 825 by separating the numbers and arranging it to ascending order. The given number is 825. ...
Final answer: The smallest 3-digit number of 825 is 258.
hope it helps
Answer:
258
Step-by-step explanation:
We are given 3 numbers:
8 2 5
And we are asked to find the smallest 3 digit number using those 3 digits above.
To make the smallest number, place the numbers in value from least to greatest:
2 5 8
This is your 3 digit number: 258.
Hope this helps! :)
A company has 10 software engineers and 6 civil engineers. In how many ways can they be seated around a round table so that no two of the civil engineers will sit together? [ 9! × 10!/4!)]
The software engineers can be seated on a round table with no two civil engineers sitting together is 9!×10!/4!
Given, a company has 10 software engineers and 6 civil engineers.
we need to determine in how many ways can they be seated around a round table so that no two civil engineers will sit together.
10 software engineers can be arranged around a round table in :
=(10-1)!
= 9! ways .... eq(A)
Now, we must arrange the civil engineers so that no two can sit next to one another. In other words, we can place 6 civil engineers in any of the 10 *-designated roles listed below.
This can be done in ¹⁰P₆ ways ...(B)
From A and B,
required number of ways = 9!×¹⁰P₆
= 9! × 10!/4!
Hence the number of ways the engineers can be seated is 9! × 10!/4!.
Learn more about Permutations and combinations here:
brainly.com/question/4658834
#SPJ1
-6-2b=-8. What's the answer
Answer: B = 1
Step-by-step explanation: The first step is to add 6 to both sides of the equation to get -2b = -2
You then divide both sides by negative 2 so you can get a positive
dividing both sides by negative 2 will leave you with 1b = 1
so
b = 1
find a word that has a value of 18. (Show calculations)
Based on the value of each letter, a word that can be found to have a value of 18 is Beef.
What word has a value of 18?The question is not complete so I'll answer it using certain assumptions to provide a model you can apply to yours.
Assuming that the value of each letter in a word is the position of the word on the alphabet then a word that has the value of 18 would be BEEF.
The value of B would be 2.
The value of E is 5.
The value of F is 6.
When these are added up:
= 2 + 5 + 5 + 6
= 18
Find out more on the using letters as values at https://brainly.com/question/17750213.
#SPJ1
100 Points! Geometry question. Photo attached. Please show as much work as possible. Thank you!
Answer: 127
Step-by-step explanation:
360 - 70 - 36 = 254
254/2 = 127
D = 127
Question 6 of 10
What is the solution to the system of equations graphed below?
y = -2x + 4
y=x-5
Answer:
the solution to the system of equations is (x, y) = (3, -2).
Step-by-step explanation:
y = -2x + 4 (Equation 1)
y = x - 5 (Equation 2)
To solve for x, we can equate the right sides of the equations:
-2x + 4 = x - 5
Now, we can solve for x:
-2x - x = -5 - 4
-3x = -9
x = (-9) / (-3)
x = 3
Substituting the value of x back into Equation 2, we can solve for y:
y = 3 - 5
y = -2
Isabel went to the grocery store. She spent $15.91 on vegetables and $11.22 on fruit. She also bought some bread. If she paid with 3 ten dollar bills and got 45 cents back in change, how much did she spend on bread?
Based on an equation, the amount that Isabel spent on bread was $2.42.
How the equation was solved?To solve the equation for the amount spent on bread, we determined the total amount Isabel spent by subtracting the change from the total bills she had.
An equation is a mathematical statement of the equality or equivalence of two or more mathematical expressions.
The amount Isabel spent on vegetables = $15.91
The amount she spent on fruits = $11.22
Let the amount she spent on bread = x
The total amount she went with = $30 (3 x $10)
The change she got after giving the cashier $30 = $0.45
The total amount Isabel spent for vegetables, fruits, and bread = $29.55 ($30.00 - $0.45)
x = $2.42 ($29.55 - $15.91 - $11.22)
Check:
Vegetables = $15.91
Fruits = $11.22
Bread = $2.42
Total expenses = $29.55
Learn more about equations at https://brainly.com/question/29174899.
#SPJ1
plzzzz... tell fast
Answer:
its 4x
Step-by-step explanation:
Fred did 600 J of work in 60 seconds. Calculate his power.
O
A. 600 W
B. 100 N
C. 500 N
OD. 10 W
Answer:
To calculate Fred's power, we can use the formula:
Power = Work / Time
Given that Fred did 600 J of work in 60 seconds, we can substitute the values into the formula:
Power = 600 J / 60 s
Power = 10 W
Therefore, the correct answer is:
D. 10 W
Step-by-step explanation:
Answer:
Power = Work/Time
Power = 600 J/60 s = 10 W
Therefore, the answer is D. 10 W.
Step-by-step explanation:
IV.Activity
Which items can you buy for 2000?
(A) Bat and Helmet
(B) Glove and Bat
(C) Bat and Pad
(D) Helmet, Glove and Pad
Answer:
bat and pad is correct answer
Ackerman and Goldsmith (2011) found that students who studied text from printed hardcopy had better test scores than students who studied from text presented on a screen. In a related study, a professor noticed that several students in a large class had purchased the e-book version of the course textbook. For the final exam, the overall average for the entire class was � = 81.7, but the n = 9 students who used e-books had a mean of M = 77.2 with a standard deviation of s = 5.7.
The sample is sufficient to conclude that scores for students using e-books were significantly different from scores for the regular class.
What are the hypothesis tested?At the null hypothesis, it is tested if the scores are not different from the regular class, that is, if the mean is of 81.7, hence:
\(H_0: \mu = 81.7\)
At the alternative hypothesis, it is tested if the scores are different from the regular class, that is, if the mean is different of 81.7, hence:
\(H_1: \mu \neq 81.7\)
We have a two-tailed test, as we are testing if the mean is different of a value, with a significance level of 0.05 and 9 - 1 = 8 df, hence the critical value is given as follows:
|t| = 2.306.
Thus the decision rule is given as follows:
|t| < 2.306 -> do not reject the null hypothesis -> sample is not sufficient evidence.|t| > 2.306 -> reject the null hypothesis -> sample is sufficient evidence.What is the test statistic?The test statistic is given by the equation presented as follows:
\(t = \frac{\overline{x} - \mu}{\frac{s}{\sqrt{n}}}\)
In which:
\(\overline{x}\) is the sample mean.\(\mu\) is the value tested at the null hypothesis.s is the standard deviation of the sample.n is the sample size.The parameters for this problem are of:
\(\overline{x} = 77.2, \mu = 81.7, s = 5.7, n = 9\)
Hence the test statistic is calculated as follows:
\(t = \frac{\overline{x} - \mu}{\frac{s}{\sqrt{n}}}\)
\(t = \frac{77.2 - 81.7}{\frac{5.7}{\sqrt{9}}}\)
t = -2.37.
|t| > 2.306, hence the sample is sufficient to conclude that scores for students using e-books were significantly different from scores for the regular class.
Missing InformationThe problem asks if the sample is sufficient to conclude that scores for students using e-books were significantly different from scores for the regular class? Use a two-tailed test with α = .05.
More can be learned about the test of an hypothesis at https://brainly.com/question/13873630
#SPJ1
Can someone help me pls pls help with this question
Answer:
D.
Step-by-step explanation:
The mean is 11,000
a charity raised about 8200. if the amount of money was rounded to the nearest hundred dollars, what ciuld have been the exact amount?
Answer:
Anything from 8150 to 8249
please help!
Jocelyn has a points card for a movie theater.
She receives 25 rewards points just for signing up.
She earns 12.5 points for each visit to the movie theater.
She needs at least 170 points for a free movie ticket.
Write and solve an inequality which can be used to determine v, the number of visits Jocelyn can make to earn her first free movie ticket.
Answer:
25 + 12.5v ≤ 170
Step-by-step explanation:
Suppose a population contains 20,000 people. All else being equal, a study
based on a population sample that includes which of the following numbers
of respondents would be the most reliable?
A. 200
OB. 20
C. 2000
D. 2
A study based on a population sample that includes 2000 respondents would be the most reliable out of the given options.
In statistical analysis, the reliability of a study depends on the representativeness and size of the sample.
A larger sample size generally provides more reliable results as it reduces the sampling error and increases the precision of the estimates.
Given that the population contains 20,000 people, we need to consider which number of respondents would yield the most reliable study.
Option A: 200 respondents
This represents only 1% of the population.
While it is better than having just 2 respondents, it may not be sufficient to accurately capture the characteristics of the entire population.
Option B: 20 respondents
This represents only 0.1% of the population.
With such a small sample size, the study would likely suffer from a high sampling error and may not provide reliable results.
Option C: 2000 respondents
This represents 10% of the population.
While it is a larger sample size compared to the previous options, it still only captures a fraction of the population.
The study may provide reasonably reliable results, but there is room for potential sampling error.
Option D: 2 respondents
This represents an extremely small sample size, accounting for only 0.01% of the population.
With such a small sample, the study would be highly susceptible to sampling bias and would likely yield unreliable results.
Based on the options provided, option C with 2000 respondents would be the most reliable study.
Although it does not include the entire population, a sample size of 2000 respondents provides a larger representation of the population and reduces the potential for sampling error.
However, it's important to note that the reliability of a study depends not only on sample size but also on the sampling method, data collection techniques, and other factors that ensure representativeness.
For similar question on population sample.
https://brainly.com/question/27859177
#SPJ8
PLEASE HELP ITS MATH THANK YOUUUU
Answer:
2inch=1 foot
Step-by-step explanation:
so 2 to 1 2:1
Adult tickets for the game cost $6 each
and student tickets cost $3 each. A total of
1040 tickets worth $5400 were sold. How
many student tickets were sold?
Answer:
280 student tickets were sold.
Step-by-step explanation:
To solve this problem, we will have to create a system of equations. The first equation will represent the number of tickets sold.
Let \(a\) = adult tickets.
Let \(s\) = student tickets.
The first equation:
\(a+s=1040\)
The second equation in our system will represent the total cost of the tickets sold:
\(6a+3s=5400\)
There are multiple ways to solve a system of equations, these include elimination, substitution, and graphing. Let's use the elimination method. By use of this method, I will distribute a \(-6\) into the first equation to eliminate the variable \(a\), to solve for \(s:\)
\(-6(a+s=1040)\)
\(6a+3s=5400\)
Distribute the \(-6\):
\(-6a-6s=-6240\)
\(6a+3s=5400\)
Combine the two equations:
\(-3s=-840\)
Divide both sides of the equation by the coefficient of \(s\), which is \(-3\):
\(s=280\)
Therefore, 280 student tickets were sold.
-
Though we have our answer at this point in the problem, let's find how many adult tickets there are so we can substitute in our solved values to check our work & solution.
Substitute \(280\) into the first initial equation:
\(a+280=1040\)
Subtract \(280\) from both sides of the equation:
\(a=760\)
Therefore, 760 adult tickets were sold.
We now know that our numbers are correct because when added, they equal the number of tickets given in the problem:
\(760+280=1040\)
-
Now we've confirmed that information, but to be extra sure, we can substitute our solved values into the second equation in the system to make sure our numbers add up to the $5400 the tickets were worth:
\(6(760)+3(280)=5400\)
\(4560+840=5400\)
Now we know that all of our answers are correct!
There is a line who's y-intercept is 8 and whose slope is 0. What is its equation in slope-intercept form?
Answer:
m=x+8 because the y intercept Is 8 it takes the place of b in the equation and there is no slope so there is no thus you will have a straight line on the 8on the y axis
Answer: m=x+8
good luck
Alice and Bob each have a certain amount of money. If Alice receives n dollars from Bob, then she will have 7 times as much money as Bob. If, on the other hand, she gives n dollars to Bob, then she will have 2 times as much money as Bob. If neither gives the other any money, what is the ratio of the amount of money Alice has to the amount Bob has?
The ratio of the amount of money Alice has to the amount Bob has is approximately 3.36 : 1.
How to find the ratio of money ?According to the problem, two equations can be set up:
If Alice receives n dollars from Bob, then she will have 7 times as much money as Bob. This gives us the equation:
A + n = 7(B - n)
If Alice gives n dollars to Bob, then she will have 2 times as much money as Bob. This gives us the second equation:
A - n = 2(B + n)
We can rearrange these equations to make them easier to solve:
8n = 7B - A
3n = A - 2B
Setting these equal to each other, since they are both equal to 24n, gives:
21B - 3A = 8A - 16B
37B = 11A
A / B = 37 / 11
= 3. 36
Find out more on ratio at https://brainly.com/question/31414597
#SPJ1
Consider the following statements:
Which of the statement(s) regarding the "limit" concept in
Mathematics is/are TRUE and which is/are FALSE?
I
A limit is a point or value that a pattern, function or sum of
a series, approaches as the number of terms in the series
increases.
II A limit is a value that a function approximates for a given
input value.
III A limit is a value we get closer and closer to, but never
quite achieve.
I: TRUE - A limit is a value that a function, pattern, or sum of a series approaches as the number of terms or input value increases.
What is a Limit in Mathematics based on the given context?II: TRUE - A limit can also be a value that a function approximates for a specific input value, especially when dealing with continuity or differentiability.
III: FALSE - Although a limit might seem like a value that is never quite achieved, it can be reached or even exceeded, depending on the function and its behavior.
Read more about math limits here:
https://brainly.com/question/30339394
#SPJ1
Darius deposits $600 into a simple interest savings account. After a year, the account balance will be $615. What is the interest rate?
Write the equation AND solve.
Answer:
savings account. After a year, the account balance will be $615. What is the interest rate?
Write the equation AND solve.
Step-by-step explanation:
savings account. After a year, the account balance will be $615. What is the interest rate?
Write the equation AND solve.
425 percent of what number of gallons is 46.75
Answer:
425% of 46.75=198.6875
Step-by-step explanation:
Surface area of the below shape
Solve all of the equations below and add all of the solutions to those equations to get your answer.
10x4x2=
8x4x2=
10x8=
(10x8)-(7x6)=
7x2x2=
6x2x2=
7x6=
Question 1-2
What is the value of a³ + b (6 + c), when a = 2, b = 3, and c = 4?
Answer:
\(\huge\boxed{\sf 38}\)
Step-by-step explanation:
Given expression:= a³ + b (6 + c)
Put a = 2, b = 3 and c = 4
= (2)³ + 3 (6 + 4)
= 8 + 3(10)
= 8 + 30
= 38\(\rule[225]{225}{2}\)
Answer:
Step-by-step explanation:
the requied answer is 38.
according to the question the value of a=2,b=3,c=4.
here,
to find the value of a³ + b (6 + c)we have to do it in steps:
step 1: solve the bracket (6+4) =10.
step 2: solve the value of a³ =8.
now put these values ,
=8+3(10)
=38.
Many doctors rely on the use of intravenous medication administration in order to achieve an immediate response of a particular drug's effects. The concentration, C, in mg/L, of a particular medication after being injected into a patient can be given by the function C(t) = −2t2 + 8t, where the time, t, is hours after injection.
Part A: What are the domain and range of the function C(t) based on the context of the problem? Show all necessary calculations. (5 points)
Part B: Graph the function to determine the greatest concentration of the medication that a patient will have in their body. (5 points)
The domain of the given function is the hours or time taken whereas the range will be the concentration and greatest concentration of the medication that a patient will have in their body is 8 mg/L.
What is Domain of a function?
Inputs are assigned outputs by functions. The collection of all potential inputs for a function is its domain. For instance, the domain of f(x)=x2 and g(x)=1/x are all real integers with the exception of x=0.
Therefore, The domain of the given function is the hours or time taken.
What is range of a function?
The collection of potential output values for a function is known as its range.
Therefore, the range of the function is concentration.
B) Graph of the function is attached and we can see from the graph that greatest concentration of the medication that a patient will have in their body is 8 mg/L.
To know more about range and domain, go to link
https://brainly.com/question/2264373
#SPJ1